Gibt es Fälle, in denen ∇⋅∭J(x′)|x−x′|dV′≠0∇⋅∭J(x′)|x−x′|dV′≠0\nabla\cdot\iiint\frac{\ mathbf{J}(\mathbf{x}')}{\left|\mathbf{x}-\mathbf{x'}\right|}\mathrm{d}V' \neq 0?

In Jacksons Classical Electrodynamics , Abschnitt 5.4 (Vektorpotential), scheint der Autor davon auszugehen, weil J = 0 , gilt für die Stromdichte (wobei über den ganzen Raum integriert wird):

J ( X ' ) | X X ' | D v ' = 0

Im Allgemeinen kennen wir das jedoch in einem geschlossenen Volumen v , wir haben:

v J ( X ' ) | X X ' | D v ' = v J ( X ' ) | X X ' | D v ' = v J ( X ' ) ( 1 | X X ' | ) D v '
= v J ( X ' ) ' ( 1 | X X ' | ) D v ' = v ' J ( X ' ) | X X ' | D v ' + v ' J ( X ' ) | X X ' | D v '

Nun im magnetostatischen Fall wissen wir aus der Kontinuitätsgleichung J + ρ T = 0 das in der Tat, J = 0 über den gesamten Raum (weil es keine lokalen Schwankungen der Ladungsdichte gibt), und so verschwindet der zweite Term und wir haben übrig:

v J ( X ' ) | X X ' | D v ' = v ' J ( X ' ) | X X ' | D v ' = v J ( X ' ) | X X ' | D S '

Wenn wir nun eine Grenze nehmen, da das Volumen zum gesamten Raum wird, sehen wir, dass das Oberflächenintegral in zunehmend weit entfernten Regionen genommen wird X . Wenn wir bestimmte Annahmen über die Asymptotik von treffen J , wie zum Beispiel | J ( X ' ) | = Ö ( 1 | X X ' | ) , dann können wir das Oberflächenintegral begrenzen und zeigen, dass es gegen Null geht.

Physikalisch könnte man argumentieren, dass dies ausreicht, da realistische Systeme sowieso immer Ströme in einem endlichen Volumen gebunden haben. Aber manchmal betrachten wir idealisierte Szenarien wie unendliche gerade Linien mit einer gleichmäßigen Strömung ICH . Diese Fälle können Probleme verursachen. Wenn Sie nur einen einzigen (oder endlich viele) solcher Drähte haben, können Sie meiner Meinung nach immer noch zeigen, dass das Integral wegen der umgekehrten Abhängigkeit vom Abstand im Integranden gegen Null geht (ich bin mir nicht sicher). Aber selbst dann könnte man sich vernünftigerweise idealisierte Situationen vorstellen, die unendlich viele solcher Drähte umfassen, die dazu führen könnten, dass das Oberflächenintegral nicht gegen Null konvergiert.

Eine weitere Schwierigkeit besteht darin, dass die Konvergenz für jede Oberfläche gelten sollte , die schließlich den gesamten Raum umschließt. Beschränken wir uns auf Radiuskugeln | X X ' | = R fokussiert auf X , dann ist die Konvergenz trivial, weil wir erhalten

v J ( X ' ) | X X ' | D S ' = 1 R v J ( X ' ) D S ' = 0
(nach dem Divergenzsatz und der Kontinuitätsgleichung). Aber wenn Ihre Oberflächen nicht kugelförmig sind, funktioniert dieser Trick nicht mehr wirklich. Aber vielleicht gibt es eine Möglichkeit, dieses Problem zu vermeiden.


Ich bin daran interessiert, die allgemeinsten Annahmen zu kennen, die gemacht werden können J um die Konvergenz zu erfüllen, und auch um Situationen zu kennen, die in Betracht gezogen werden könnten, in denen diese Annahme falsch ist.

J = 0 impliziert einen stationären Zustand. Und das folgende Intergal ist über den Quellen - es ist nicht über dem gesamten Raum: J ( X ' ) | X X ' | D v ' . Es gibt so viele Cluster aus den dreifachen Integralen, dass es für mich schwer zu lesen ist.
Nun, es ist über allem Raum, in dem Sinne, dass man überall dort, wo es keine Quellen gibt, einfach etwas hat J ( X ' ) = 0 . Wie ich im Hauptteil der Frage erwähnt habe, können Sie manchmal Szenarien in Betracht ziehen, in denen J in einem unbegrenzten Raumbereich ungleich Null ist. Es tut mir leid für das Durcheinander, das durch die Gleichungen verursacht wurde, vielleicht habe ich zu viele Details angegeben. Aber ich versuche normalerweise, eher zu viel als zu wenig zu sagen.
Irgendetwas stimmt mit der dritten Zeile nicht. Umschreiben: ' J ( X ' ) | X X ' | D v ' als D F J D v - Weglassen des Minuszeichens, der x-Abhängigkeiten und der Primzahlen - wo F = 1 / R .Dann nach dem Divergenzsatz: D F J D v = D F J N ^ D σ D F J D v . Wenn J = 0 dann das andere 2 Integrale sind untereinander gleich. Wenn das Oberflächenintegral verschwindet, verschwindet auch die 3 R D Integral.

Antworten (3)

Warum behauptest du das

v J ( X ' ) | X X ' | D v ' = v ' J ( X ' ) | X X ' | D v '
? Das scheint nicht zu stimmen. Du solltest haben
v J ( X ' ) | X X ' | D v ' = v ( 1 | X X ' | ) J ( X ' ) D v ' = = v ( ' 1 | X X ' | ) J ( X ' ) D v ' = = v 1 | X X ' | ' J ( X ' ) D v ' v ' ( 1 | X X ' | J ( X ' ) ) D v ' = = 0 v 1 | X X ' | J ( X ' ) D S
und dies verschwindet, wenn J verschwindet schnell genug. Beachten Sie, dass Sie möglicherweise Probleme mit der Konvergenz des Integrals haben, wenn es nicht schnell genug verschwindet J ( X ' ) | X X ' | D v ' den ganzen Raum übernommen; Um sicherzustellen, dass es gut definiert ist, benötigen Sie J schnell genug verschwinden, und das lässt den Grenzterm im Unendlichen verschwinden.

Sie haben Recht, ich glaube, ich habe bei dieser ersten Gleichheit einen Fehler gemacht, und ich habe ihn jetzt korrigiert. Zum Glück haben wir am Ende das gleiche Oberflächenintegral, weil beide J ( X ' ) = 0 Und ' J ( X ' ) = 0 .
Meine Frage, ob das Oberflächenintegral nicht immer Null ist, bleibt jedoch bestehen. Das weiß ich wenn J schnell genug verschwindet, ist das Integral Null. Aber es gibt Fälle, auf die ich hingewiesen habe, in denen dies nicht zutrifft (z. B. unendliche Drähte oder vielleicht eine Idealisierung, bei der über den gesamten Raum eine gleichmäßige Stromdichte herrscht). In diesem Fall ist es auch nicht klar, dass das Integral divergiert, aber ich weiß nicht, wie ich es zeigen soll.

Ihre Idee zu Radiuskugeln R gibt den Hinweis; wenn sich die Stromdichte gut verhält, nimmt der große Abstand von der Oberfläche ab X macht das Integral, in der Grenze R , auf Null gehen.

Die betrachtete Situation ist, dass elektrischer Strom fließt, ohne dass sich Ladungen ansammeln; was in eine region hineingeht, geht auch aus der region hinaus. Dies ist in der Praxis sehr üblich, der gesamte eingehende Strom entspricht dem gesamten ausgehenden Strom. In der Praxis ist dieser eingehende Strom begrenzt, egal wie die Grenzfläche gewählt wird, weil der Strom im einzelnen Draht endlich ist und die Anzahl der Drähte endlich ist.

Das fragliche Integral kann in zwei Teile unterteilt werden, einen aufgrund des Stroms, der in die Region hineingeht, und einen aufgrund des Stroms, der aus der Region hinausgeht:

v J / R D S = C ich N + C Ö u T
Wo
C ich N = v χ ich N ( X ' ) J ( X ' ) / R D 2 X '
Und
C Ö u T = v χ Ö u T ( X ' ) J ( X ' ) / R D 2 X '
Wo χ ich N ist eine charakteristische Funktion des Teils der Oberfläche, in den der Strom einfließt ( J D S < 0 ).

Wenden wir die Dreiecksungleichung an:

| v J / R D S | | C ich N | + | C Ö u T | .

Das Integral geht also offensichtlich gegen Null, wenn beides der Fall ist C ich N Und C Ö u T auf null gehen. Dass diese beiden auf Null gehen, ist eine ausreichende Bedingung (es ist möglicherweise keine notwendige).

Lassen Sie die Kleinsten R = | X X ' | für irgendein Stadium des Begrenzungsprozesses bezeichnet werden R M ich N . Natürlich muss sich beim Begrenzungsprozess die gesamte Grenze ins Unendliche ausdehnen, also R M ich N .

| C ich N | v χ ich N ( X ' ) | J ( X ' ) D 2 X ' | / R M ich N = ICH ich N R M ich N
Wo ICH ich N ist (positiver) Wert des Stroms aufgrund von Ladungen, die in die Region gelangen. Solange dieser Strom nicht zu schnell mit anwächst R M ich N , geht der Beitrag gegen Null, wenn die Grenze ins Unendliche erweitert wird. Ebenso für den anderen Beitrag.

Die ausreichende Bedingung dafür, dass das Oberflächenintegral auf Null geht, ist also, dass der elektrische Strom, der durch die Oberfläche fließt, nicht zu schnell wächst, wenn sich die Oberfläche ausdehnt. Wenn der Strom unabhängig von der Grenze durch einen bekannten Maximalwert begrenzt ist, wie es der Fall ist, wenn das System aus einer endlichen Anzahl von (möglicherweise unendlich langen) Drähten mit endlichem Strom besteht, dann geht das Integral gegen Null. Somit kann man eine beliebige endliche Anzahl von unendlichen Drähten betrachten, die jeweils einen endlichen Strom führen. Wenn jedoch die Anzahl der Drähte, die die Grenze überqueren, ebenso schnell oder schneller als zunimmt R M ich N , dann könnte es ein Problem geben und das Integral hat möglicherweise nicht den Grenzwert 0. Dies sieht jedoch nicht nach einer üblichen Situation aus.

Das sieht wirklich nach einem guten Argument aus. Ich mag die Idee, die Einwärts- und Auswärtsströme beim Begrenzen des Oberflächenintegrals zu trennen, da ich denke, dass Sie dadurch bessere Grenzen für die Skalarprodukte erhalten. Ich stimme zu, dass dies möglicherweise keine notwendige Bedingung ist, weil wir die Möglichkeit der Aufhebung zwischen den inneren und äußeren Strömungen bei der Integration nicht voll ausschöpfen.
Nun, eigentlich jetzt, wo ich darüber nachdenke, könntest du das nicht ersetzen | C ich N | + | C Ö u T | gebunden | ( | C ich N | | C Ö u T | ) | weil wir bereits wissen, dass sie entgegengesetzte Vorzeichen haben? Jetzt können wir das Integral sogar dann auf Null gehen lassen, wenn ICH ich N / R M ich N geht nicht zu 0 , so lange wie ICH ich N Und ICH Ö u T liegen sehr nahe beieinander.
Ich glaube nicht, wegen des Faktors 1 / R , C's unterscheiden sich von ICH 'S; C sind einander nicht entgegengesetzt und haben nicht notwendigerweise entgegengesetzte Vorzeichen. Sie müssen separat eingeschränkt werden.
Ich denke, wir wissen es C ich N Und C Ö u T haben entgegengesetzte Vorzeichen, gegeben Ihrer Definition, weil das Skalarprodukt J D S ' ist immer positiv für C Ö u T und negativ für C ich N .
Das wissen wir also C ich N 0 Und C Ö u T 0 . Wir wissen es auch ICH ich N / R M A X C ich N ICH ich N / R M ich N Und ICH Ö u T / R M A X C Ö u T ICH Ö u T / R M ich N . Das Oberflächenintegral ist also beschränkt durch C Ö u T + C ich N | | C Ö u T | | C ich N | | max ( ICH Ö u T / R M ich N ICH ich N / R M A X , ICH Ö u T / R M A X ICH ich N / R M ich N ) .
Aufgrund der J = 0 Zustand, den wir immer haben sollten ICH ich N = ICH Ö u T = ICH (für die gleiche Oberfläche). Ich denke also, wir können die obige Schranke wie folgt vereinfachen C Ö u T + C ich N ICH ( 1 / R M ich N 1 / R M A X ) . Ja, diese Grenze ist nicht so gut, wie ich anfangs dachte, denke ich. Der 1 / R M A X Teil kann beliebig klein sein, daher hilft es im Vergleich zu Just normalerweise nicht allzu viel ICH / R M ich N .
Ich vermute jedoch, dass es eine bessere Grenze geben könnte, da die Oberflächen geschlossen und wahrscheinlich kontinuierlich sind, sodass wir Einschränkungen hinsichtlich der möglichen Geometrie haben. Insbesondere kannst du nicht nur Strom nach innen an haben R M ich N und Strom nach außen bei R M A X Es wird also einen Übergangsbereich geben, in dem R M ich N < R < R M A X . Ich bin mir nicht einmal sicher, ob es explizite Beispiele gibt, bei denen das Oberflächenintegral wirklich divergiert (oder nicht geht 0 ). Vielleicht so etwas wie eine gleichmäßige Stromdichte J ( X ) = J 0 z ^ ? Aber wir müssen auch sicherstellen, dass das ursprüngliche Volumenintegral konvergiert.

Ich habe eher wenige Gedanken als eine Antwort. Vielleicht hilft es. Da Ihre Stromdichte divergent ist, ist es vielleicht eine gute Idee, die Helmholtz-Zerlegung anzuwenden?

Für ein generisches, gut erzogenes 3D-Vektorfeld mit . J = 0 wir haben:

J ( R ) = × M

Für irgendein Vektorfeld M , aber dann (unter Verwendung des Levi-Civita-Symbols ϵ a β γ um mit Locken umzugehen):

D 3 R ' J ( R ' ) . ' 1 | R R ' | = ϵ a β γ D 3 R ' β ' M γ ( R ' ) . a ' 1 | R R ' | = ϵ a β γ D 2 R ' N ^ β M γ ( R ' ) . a ' 1 | R R ' |

Wo N ^ ist die Normale zur Oberfläche des Volumens, über das Sie integrieren, und ich habe die Symmetrie von Levi-Civita verwendet, um das andere Integral loszuwerden.

Hat das geholfen? Nun, wir haben immer noch ein Oberflächenintegral, aber jetzt sprechen wir über Magnetisierung ( M ), die nicht so schnell verschwinden muss, damit das Integral gegen Null konvergiert. Sie können den gleichen Trick auch wiederholen, tatsächlich geht keine Allgemeingültigkeit verloren, wenn Sie dies annehmen . M = 0 . Ich frage mich, ob Sie einen iterativen Beweis erstellen können, bei dem Sie immer tiefer in die Ableitungen der Stromdichte einsteigen, der dann feststellt, dass das anfängliche Integral so nahe an Null gebracht werden kann, wie Sie möchten.